Logical equivalence of statements with truth tables

  • Thread starter Thread starter icantadd
  • Start date Start date
  • Tags Tags
    Equivalence
Click For Summary
The discussion centers on determining the logical equivalence of two statements: (S ∧ T) ∧ ¬(S ∧ T) and (S ∨ T) ⇒ (S ∧ T). It is clarified that logical equivalence means two predicates yield the same output for the same inputs. The truth tables indicate that the first statement is always false, while the second can be true if both S and T are true. Therefore, the two statements are not logically equivalent, as they do not produce the same truth values across all combinations of S and T.
icantadd
Messages
109
Reaction score
0

Homework Statement



(S \wedge T) \wedge \neg (S \wedge T)

(S \vee T) \Rightarrow (S \wedge T)


Are the two (predicates?) logically equivalent?

Homework Equations



Not sure, but I believe that logical equivalence means that two predicates give the same output on the same input.

The Attempt at a Solution



Worked truth tables. The end result of both where statement T is t t f f and statement S is t f t f, in both cases is t f f t. I guess I am not really sure if that means that the two predicates are logically equivalent or not. I was actually just playing with truth tables and got this on accident, and wasn't sure if the fact that they both have the same end result is worth noting or not.
 
Physics news on Phys.org
icantadd said:
Not sure, but I believe that logical equivalence means that two predicates give the same output on the same input.

Correct.

I will take a look at this in the afternoon if other PF'ers haven't got to it before me...its 2.30 in the morning.
 
icantadd said:

Homework Statement



(S \wedge T) \wedge \neg (S \wedge T)

(S \vee T) \Rightarrow (S \wedge T)


Are the two (predicates?) logically equivalent?

Homework Equations



Not sure, but I believe that logical equivalence means that two predicates give the same output on the same input.

The Attempt at a Solution



Worked truth tables. The end result of both where statement T is t t f f and statement S is t f t f, in both cases is t f f t. I guess I am not really sure if that means that the two predicates are logically equivalent or not. I was actually just playing with truth tables and got this on accident, and wasn't sure if the fact that they both have the same end result is worth noting or not.

The way I see it is you just work the logic tables look at all possibilities for (S,T) i.e. (True, True), (True, False), (False, True), (False, False). If that's what you did and got the same result then I would believe they are logically equivalent.
 
icantadd said:

Homework Statement



(S \wedge T) \wedge \neg (S \wedge T)

(S \vee T) \Rightarrow (S \wedge T)


Are the two (predicates?) logically equivalent?

Homework Equations



Not sure, but I believe that logical equivalence means that two predicates give the same output on the same input.
Yes, that is correct. If this is homework, surely you could look that up in your textbook?

The Attempt at a Solution



Worked truth tables. The end result of both where statement T is t t f f and statement S is t f t f, in both cases is t f f t. I guess I am not really sure if that means that the two predicates are logically equivalent or not. I was actually just playing with truth tables and got this on accident, and wasn't sure if the fact that they both have the same end result is worth noting or not.
You just said " logical equivalence means that two predicates give the same output on the same input"
 
First, note that

<br /> (S \wedge T) \wedge \neg (S \wedge T)<br />

will always be false (for every combination of S and T) since (S \wedge T) and \neg (S \wedge T) always have opposite truth values.

However, if both S and T are true, then so is

<br /> (S \vee T) \Rightarrow (S \wedge T) <br />

These two statements are not equivalent.
 
Question: A clock's minute hand has length 4 and its hour hand has length 3. What is the distance between the tips at the moment when it is increasing most rapidly?(Putnam Exam Question) Answer: Making assumption that both the hands moves at constant angular velocities, the answer is ## \sqrt{7} .## But don't you think this assumption is somewhat doubtful and wrong?

Similar threads

  • · Replies 2 ·
Replies
2
Views
2K
  • · Replies 12 ·
Replies
12
Views
1K
  • · Replies 6 ·
Replies
6
Views
3K
Replies
3
Views
1K
  • · Replies 4 ·
Replies
4
Views
2K
  • · Replies 6 ·
Replies
6
Views
2K
  • · Replies 2 ·
Replies
2
Views
2K
Replies
20
Views
2K
  • · Replies 2 ·
Replies
2
Views
2K
Replies
3
Views
2K